What are your chances of acceptance?

Calculate for all schools, your chance of acceptance.

Duke University

Your chancing factors

Extracurriculars.

how to write a thesis statement for apush dbq

How to Write the Document Based Question (DBQ)

Do you know how to improve your profile for college applications.

See how your profile ranks among thousands of other students using CollegeVine. Calculate your chances at your dream schools and learn what areas you need to improve right now — it only takes 3 minutes and it's 100% free.

Show me what areas I need to improve

What’s Covered:

What is the document based question, steps to writing an effective dbq, how do ap scores affect my college chances.

If you’re taking a history AP exam, you’ll likely encounter the Document Based Question (DBQ). This essay question constitutes a significant portion of your exam, so it’s important that you have a good grasp on how best to approach the DBQ. In this post, we’ll cover what exactly a document based question is, and how to answer it successfully.

A Document Based Question (DBQ) is a measure of the skills you learned in your AP classes in regard to recalling history and analyzing related documents. These documents can be primary or secondary sources, and your responses are expected to be in the form of an essay. Your ability to relate the context of documents to concepts beyond the given text and creating meaningful connections between all your sources will help demonstrate your skills as a knowledgeable writer.

The number of documents for a DBQ varies from exam to exam, but typically will fall between five to seven documents. The following AP exams will require you to write a DBQ:

AP U.S. History

AP European History

AP World History

We’ve listed the formats for each exam below, and keep in mind that the number of documents is prone to changing from year to year:

  • Up to seven Documents
  • One hour recommended time (includes 15-minute reading period)
  • Up to seven Documents 
  • 25% of total exam score

With that in mind, let’s jump right into how to craft a strong DBQ response!

We’ve summarized how to write an effective DBQ into the following five steps:

1. Read the prompt first

Though you may be tempted to jump into the documents right away, it’s very important that you first look at what exactly the prompt is asking for. This way, when you eventually look at the documents, your focus will be narrower. A DBQ tests your reading comprehension and analysis skills more than the content itself, making it very important to understand your prompt thoroughly.

2. Skim the document titles

Each document will contain vital information regarding the context, and it’s important to scout key words regarding dates, authors, and anything pertaining to the general sense of what the documents are about. Skimming through your documents like this could save time and allow you to form a more structurally sound thesis.

Let’s take a look at the following graph and figure out how to skim the figure:

how to write a thesis statement for apush dbq

This document was in a real exam from the AP World History free response questions in 2019. It’s important to pay attention to data provided and what context can be drawn from it. In this case, we’re provided with a graph that displays the life expectancy of a country in relation to the GDP per capita of said country. Being able to skim this graph and notice the common trends in the data points could provide convenient information into the context of the document, without any further intensive reading. 

For example, seeing how countries with a GDP below 4,000 to 5,000 have lower life expectancies already gives us a potential correlation between the two factors. We can use this information to start formulating a thesis, depending on what the prompt is specifically asking for.

Remember, just skim! Don’t worry about reading the entire document yet; this strategy can keep you calm and level-headed before tackling the rest of the document. Methods like this can make acing the AP World History DBQ less intimidating! 

3. Formulate a tentative thesis

A thesis is a statement that should be proved and discussed upon. It’s important to have a strong thesis as the foundation of your DBQ, as it guides the rest of your response in relation to the context. Understanding the difference between weak and strong theses will be imperative to your success, so here is an example of a weak thesis:

“The Cold War originated from some scenarios of conflict between Soviets and some groups of oppressors.” 

Such a thesis can be considered weak for its lack of specificity, focal point, and usability as a constructive tool to write further detail on the subject. This thesis does not take a clear stance or communicate to the reader what the essay will specifically focus on. Here’s how the same thesis can be restructured to be stronger and more useful:

“The Cold War originated from tense diplomatic conflicts relating to propaganda and conspiratorial warfare between the United States and the Soviet Union.”

The information that’s been included into the second thesis about the two groups involved with the Cold War gives you more room to build a structured essay response. In relation to the rubric/grading schema for this DBQ, forming a structurally sound thesis or claim is one of the seven attainable points. Being able to contextualize, analyze, and reason off of this thesis alone could provide for two to four points – this means that five out of seven of your points revolve around your thesis, so make sure that it’s strong! Doing all of this in your fifteen minute reading period is crucial as once this is set, writing your actual response will be much easier!

4. Actively read the documents

Simply reading a document doesn’t normally suffice for creating a well-written and comprehensive response. You should focus on implementing your active reading skills, as this will make a huge difference as to how efficient you are during your work process. 

Active reading refers to reading with an intention to grab key words and fragments of important information, usually gone about by highlighting and separating important phrases. Annotations, underlining, and circling are all great ways to filter out important information from irrelevant text in the documents. 

An example of where you might find important information via active reading is the description. Circle important names or dates to contextualize the document. If you still can’t find contextual value from the title, that’s totally fine! Just scope out the rest of the document in relevance to your thesis – that is, pinpoint the specific information or text that best supports your argument. Finding one or two solid points of interest from one document is usually enough to write about and expand upon within your essay. 

how to write a thesis statement for apush dbq

Discover your chances at hundreds of schools

Our free chancing engine takes into account your history, background, test scores, and extracurricular activities to show you your real chances of admission—and how to improve them.

5. Make an Outline 

If you like outlines, making one before writing your essay might prove helpful, just be aware of the time limit and act accordingly. 

Start with your introduction, then work on the rest of your essay. This way, you can make sure your thesis is clear and strong, and it will help the graders form a clear view on what the general consensus of your paper is. Make sure to include evidence with your thesis within each paragraph and cite only relevant information, otherwise your citations could come across as filler as opposed to useful content. Every commentary or point you make should be tied in some way to the documents.

Format each body paragraph and organize your essay in a way that makes sense to you! The graders aren’t really looking at the structure of your essay; rather, they want to see that you analyzed the documents in a way that is supportive of your essay. As long as you have content from the documents which prove your thesis, the order or manner in which you present them doesn’t matter too much. What’s more important is that your essay is clear and comprehensive. As you write practice DBQs, try having someone else read your essays to make sure that the format is easy to follow.

Keep all these key details in mind as you construct your own DBQ response, and you’re well on your way to writing an effective essay!

Your chances of admission are actually not really impacted by your AP scores; however, the AP classes you take are more important than the exam scores themselves, meaning the impact of your AP scores isn’t as big as you think . 

Instead, focusing on the AP classes on your transcript and the relevance of those classes to your future major is more impactful. For a further detailed understanding of the role AP classes play in regards to your college admissions, use CollegeVine’s free Admissions Calculator , which takes into account your GPA, standardized test scores, and more. 

Additional Information

To dive deeper into DBQs, AP classes, and learning how to tackle each exam check out other resources at CollegeVine:

  • Acing the Document Based Question on the AP US History Exam
  • Acing the AP World History Document Based Question
  • Ultimate Guide to the AP U.S. History Exam
  • Ultimate Guide to the AP European History Exam
  • Ultimate Guide to the AP World History Exam

Related CollegeVine Blog Posts

how to write a thesis statement for apush dbq

WHAP Website Logo.png

Document-Based Question (DBQ)

What is a dbq, how to read the documents:, written documents, how to answer the prompt:, compare & contrast, cause & effect, change & continuity over time, how to earn all 7 points:.

Contextualization

Analysis & Sourcing

How to start writing the dbq, how to write a dbq:.

Attached below is a worksheet with an outline organizer for your DBQ. When practicing for your DBQ, feel free to download & print this to use:

iconfinder_10_171505 (1).png

7 documents

pngfind.com-documents-png-6607266 (1).pn

You are given 7 documents, and you are given a prompt, similar to an LEQ prompt. You need to write an essay, responding to the prompt, using evidence from the documents. ​You have 60 minutes in total, but of those 15 minutes are recommended for reading. The sections below describe the types of documents, types of prompts, and the rubric and how to earn each point. 

You are given 7 documents. The different types are described below:

Excerpt / Written Document

document-icon.png

Graphic, Diagram, Map, Cartoon

chart-646 (1).png

General Tips

Look at the sourcing before you read each doc to get an idea of what the doc might say

Write a quick summary (~3 bullet points) to summarize the content of each doc

Write a note of how each doc fits in with the prompt

Does it support or refute your thesis?

Which side of the prompt does it cover?

Which aspect (which body paragraph) of your prompt / thesis does it cover?

Any document with written paragraphs

Newspaper, letter, speech, historian's interpretation, constitution, religious text, etc.​

Special tips:

Before you read, read the sourcing & title and try to get an idea of what the doc might say

Take your time to understand the content of the doc; no need to rush​​

Write a few notes summarizing the doc

Figure out how the doc relates to the prompt

Does it argue one side or another?

Does it provide evidence for a specific geographical region?

Does it refute your thesis?

Which sub-category of the prompt does it answer?

Any document that is a photo

Any photo that a photographer might take, or an artist's depiction of a historical event

NOT a diagram, map, or something manmade or designed by historians

Read the sourcing & title to try to figure out what the photo might depict

Look at all aspects of the photo, get an idea of what it depicts

Does it represent a historical development?​

Does it represent an artistic movement?

Look for all signs of bias in the photo

Is it depicting a specific point of view?

Does it portray a certain culture as superior?

Does it portray a certain culture as inferior?​

Does it represent a military victory?​​​

This would mean one side is better than the other​

Does it portray something as bigger or exaggerated?

Means that the exaggerated thing is depicted as superior

Does it portray something as smaller?

Means that the thing that's depicted smaller is portrayed as inferior

Based on the point of view (bias) and the content, figure out how it relates to the prompt

Does it support / refute your thesis?

What aspect of the prompt does it answer?​

Any document that is a man-made photo

Graphic, diagram, political cartoon, map, etc.

Before you read, read the sourcing & title and try to get an idea of what the doc might depict

Look at the doc and try to figure out what it represents, or what topic it depicts

Think about the bias or point of view of the doc:

Does it represent the views of one side or another?

Is it depicting one side as exaggerated or superior to another?

Cartoons are generally biased

If it's a map, what is it representing?

Is it representing the map of industrial factories, trade routes, westward expansion, deciphered wind patterns, etc.?​

Once you figure this out, understand the historical context of the map

If it's a graphic or a diagram, what information does it detail?

If it's a population growth map, what allowed for population growth?​

Think of what the diagram depicts, and what allowed for that, and what's the historical context of that historical development?

Involves comparing & contrasting 2 different things

Most important thing is the argument: Not what the differences/similarities were, but HOW THEY WERE SIGNIFICANT

How to use the documents:

Some docs might explain the features of one of the comparand (the thing you compare), other docs may explain the other comparand

Some docs might cover both comparands

Figure out what the docs are saying for each comparand, and write your thesis based on that

What are they saying are similar & different about the 2?​

Involves examining what a certain historical development, and what were its causes & effects

What's more important is examining the significance of the causes, or how one cause/effect was more important than other causes/effects

Generally, 2 causes and 1 or 2 effects

Some docs might explain the event

Some docs might explain the causes, others might explain the effects

Draft a thesis based on the info about the causes & effects mentioned in the docs

Try to mention which causes were more significant than other causes

Involves examining what changed & what remained the same as a result of one event​

Some docs might explain the catalyst (the event that caused the changes/continuities) you write about

Some docs might describe the changes

Some docs might describe the continuities

Draft a thesis based on what the docs say about the changes & continuities

How to Earn all 7  Points:

Contextualization (1 point).

Examine the historical context of the story

Kind of like a "recap" or a "flashback"

Like at the beginning of a TV show, it shows a recap of the previous episode

How to write one

Always include the time period & possibly the location​

"In Europe in the period 1450 - 1750, ..."

Provide a brief 3-5 sentence recap of how the world arrived at the situation you are writing about in your essay

The contextualization should finish with how the world arrived at the historical development you write about in your thesis, so that there is a smooth transition from contextualization to the thesis

Sample Contextualizations

Topic: Related to the industrial revolution​

Before the 1750s, people were performing manual labor, making items by hand, which was very inefficient. From 1750-1900, Europe and the rest of the world underwent an economic transformation called the Industrial Revolution. Starting in Britain due to its abundance of raw materials & strong financial support, industrial capitalists built factories powered by waterwheels or coal that artificially produced goods such as textiles, eliminating the need to make them by hand. This brought a lot more people from the countryside to the cities, where they worked in factories for low wages. From Britain, the industrial revolution spread throughout Europe as well as to US, Egypt, Russia, and Japan. [Insert Thesis Here]

Topic: Related to imperialism​

In the period 1750-1900, Europe underwent an economic transformation known as the industrial revolution, where people would use artificial power to cheaply & efficiently manufacture goods in commercial factories in the cities, rather than making goods by hand at home. In order for these factories to produce goods, they needed raw materials, which is why they had to look to other nations like those in Africa and Asia to supply raw materials to them. Thi​s led to European imperialism, a development where Europeans started colonizing other nations throughout the world, especially in Africa and Asia, to establish export-oriented economies to get raw materials to supply to their factories. [Insert Thesis Here]

Thesis (1 point)

This is your argument

Must be something that can be opposed​

Someone else has to be able to write an essay whose thesis is the opposite of yours

Must contain an argument, and generally 2-3 examples (topics for body paragraphs)

Better to have a concession

Useful for complexity point

Format of Thesis & Examples

Color Key: 

Concession / Counterargument*

Similarities / Continuities / Causes

Difference for Comparand 1 / Changes / Effects

Differences for Comparand 2

*Concession is always optional. Described in the analysis section, it can be used to get the extra complexity point

Prompt: Compare & Contrast

Although some may believe [counterargument]* , w hile [comparand 1] and [comparand 2] both [insert similarities] , [comparand 1] was [insert difference for comparand 1], and [comparand 2] was [insert difference for comparand 2], which [is why / allowed for] [insert argument]. 

Although the Delhi Sultanate had very strict religious authority, while the Delhi Sultanate and the Chola Kingdom both used religion to maintain stability , the Delhi Sultanate was attempting to impose Islam on a Hindu-majority population , and the Chola Kingdom imposed Hindu on a Hindu population , which allowed for the Chola Kingdom to be more successful than the Delhi Sultanate. 

Prompt: Change & Continuity over Time

Although some may believe [counterargument]* , as a result of [catalyst],  while [continuity] stayed the same , [change] changed,   which [is why / allowed for] [insert argument]. 

Although some may believe the Catholic church actually became more powerful, as a result of the Protestant Reformation, while women still maintained strictly subordinate roles , there were more religious wars, and more monarchs were able to consolidate more power for themselves , which caused the Catholic church to decline in power.  

Prompt: Cause & Effect

Although some may believe [counterargument]* are the most important causes of [event] ,  [causes] were the main causes** , which caused [effects].  

Although some may believe that the desire to spread Christianity was the main cause of European imperialism , the desire to get raw materials and the need for more markets were the main causes , which led to a more integrated global economy and the development of technological infrastructure in the colonies. 

**Here, the argument is that the causes you described in the blue section are more important than the causes in the yellow section. There is no need for an extra argument at the end

Evidence (3 points)

This is where you put examples / pieces of evidence to support your thesis​

To get 1 point : Use evidence from 3 of the docs

To get 2 points : Use evidence from 6 of the docs, and put an extra analysis to connect it to the thesis

It's always better to use all 7 docs in case you use one incorrectly

To get 3 points : Use an extra piece of evidence (from your own knowledge, not from the docs), and put an extra analysis to connect it to the thesis

Examples of How to Write your Evidence

How to earn the first point:

To earn the 1st point, you need to describe / state evidence from 3 docs without connecting it to the thesis

According to document 3, the Chola Empire used Hinduism as the state religion. 

According to document 7, there were more factories in Britain than in France. 

How to earn the second point:

To earn the 2nd point, you need evidence from 6* docs, and you also need to connect it to the thesis

According to document 3, the Chola Empire used Hinduism as the state religion. Because the population was also mostly Hindu, the Chola Empire was able to maintain stability by using a common belief in Hinduism to stabilize its rule. 

According to document 7, there were more factories in Britain than in France. Thus, Britain had a larger industrial output than France, which is why it was able to manufacture more weapons during World War 1 and why France relied on Britain for support. 

*Always use all 7 docs to in case you use one doc incorrectly

How to earn the third point:

To earn the third point, you need to include one piece of evidence that is not in the documents and is from your own knowledge. 

Think of what evidence or what viewpoint is missing

If it's a compare & contrast: is there any other similarity or difference? Do you have any other evidence to support the topics of your thesis?

If it's a change & continuity over time: Is there any other evidence to support one of your changes or continuities?

If it's a cause & effect: Is there any other evidence or historical content that can support your causes or effects?

Analysis (2 points)

This is the hardest part

For 1 point, you need to explain how the source of 3 documents affects either your argument or what the document has to say

There are 4 parts of sourcing, and you ONLY NEED TO CHOOSE ONE

Explained in more detail below​

For the 2nd point, you need to use complex analysis in your argument

This is the most confusing

The easiest way is to weave a counterargument through your essay, which the concession already sets you up for

The best way is to not think about it too much and just put a bit more complex arguments into your essay rather than sticking to a strict format

Historical Context

Point of View

You need to choose ONE of the above and follow the instructions below. Each of the sections below has information about each aspect of sourcing. 

You need to do this for THREE different sources to earn full points (we recommend you do 4 in case one is wrong)

Historical Context:

Explain how the historical context of any document affects what the document argues

This document was written after WW1 when everyone was feeling depressed and economically poor, which explains why the priest is talking about a revival of religion and cheerful spirits. 

This document was written in a time after the Protestant Reformation when there were a lot of religious wars, which is why the document argues that Lutheranism is better than Calvinism. 

Explain how the intended audience of any document affects what the document argues

This document was written for the Armenians of the Ottoman empire, a Christian minority that was believed to conspire with the Allies, which is why the document is very aggressive toward them in asserting Ottoman dominance. 

This speech was written to the American people to gain support for the Treaty of Versailles, which is why it intends to boost nationalist sentiment and promote American power. 

Explain how the purpose of any document affects what the document argues

This speech was written by the Republic party with the purpose of convincing its audience to vote for them, which is why it argues that Free Silver, a democratic idea, is bad. 

This speech was written by John of Montecorvino, the Archbishop of Khanbaliq who sought to convert the Mongol boys to Christianity, which is why he emphasizes how Christianity allows one to achieve salvation. 

Point of View:

Explain how the point of view of any document affects what the document argues

This speech was written from the point of view of an Indian cotton farmer, which is why he writes that the British completely destroyed the Indian handmade textile industry. 

This document was written from the point of view of Grover Cleveland, an anti-imperialist president, which is why he writes about the harms of annexing Hawaii. 

The Complexity Point

The final point is the complexity point. This is given if you have a complex argument, and it is hard to achieve. The best way to think about this is do more than the prompt asks, and add a bit of extra analysis into the essay. 

The easiest way to do this is weave a counterargument through the essay. In our thesis samples above, we already set you up for this with our concession clause. 

How to Start Writing the DBQ:

First step is to outline your essay. Follow the steps below:

Read through each document, write a brief summary, and figure out how it relates to the prompt (which side/aspect does it argue?)​

Write your thesis. Write each aspect of the thesis (concession/counterargument, evidence 1, evidence 2, argument), and combine them

Write the outline for your body paragraphs. Write the topic for each body paragraph, and which docs you'll use in each. Also, denote where you'll use your outside evidence

Write an outline for your sourcing. Choose 4 different documents, and write the sourcing sentence following the guidelines in the sourcing section above

Start writing. Good luck!

how to write a thesis statement for apush dbq

  • AP Calculus
  • AP Chemistry
  • AP U.S. History
  • AP World History
  • Free AP Practice Questions
  • AP Exam Prep

APUSH DBQ Strategies

How to Approach Document-Based AP U.S. History Questions

Question 1 in Section II of the AP US History Exam is the document-based question (DBQ). It will always include seven documents offering a variety of perspectives on a historical development or process that took place between the years 1754 and 1980. A high-scoring DBQ response will do the following.

  • Thesis: Make a thesis or claim that responds to the prompt. The thesis or claim must be historically defensible and establish a line of reasoning.
  • Context: Provide context relevant to the prompt by describing a broader historical development or process.
  • Evidence: Use at least six of the provided documents to support an argument in response to the prompt.
  • Additional Evidence: Use a historical example not found in the documents as evidence relevant to an argument about the prompt.
  • Sourcing: Explain how the context or situation of at least three documents is relevant to an argument. This could address the relevance of the document’s point of view, purpose, historical situation, and/or audience.
  • Complex Understanding: Demonstrate a nuanced understanding of an argument that responds to the prompt by using evidence to corroborate, qualify, or modify the argument.

While this may sound like a lot factors to keep in mind, the strategies below will help you plan your response in such a way to address all the scoring requirements.

AP US History DBQ Strategy

During step 1: analyze the prompt.

  • Use the 15-minute reading period to read the documents and organize them into groups for analysis.
  • Feel free to write notes in the test booklet and underline important words in both the source line and the document itself. Nothing in the booklet is read as part of the essay scoring.
  • Assume that each document provides only a snapshot of the topic—just one perspective.
  • For each document, jot down brief notes to solidify your understanding of the document. The notes will also help you quickly review the documents when you make your plan and write your essay. Take short notes about: the main idea(s) of the source, the purpose of the source (why it was written), and the background of the author and/or the context in which the source was created. Thinking about these factors will help you address the DQB requirements.
  • Reread the prompt, thinking about how each of the documents relates to the prompt. Group the documents by their similarities: perhaps they present two or more major viewpoints or can be grouped according to two or more types of changes.
  • If the 15-minute reading period has passed and you need a few more minutes to review the documents and organize your thoughts, go ahead! The 15 minutes is a suggested amount of time. That said, you will want to give yourself as much time as possible to write a thoughtful response.

During Step 2: Plan Your Response

  • Making a careful plan can help you make sure you address all the scoring requirements.
  • Paraphrase your thesis statement. Knowing your claim will make it easier for you to plan an effective argument in your essay. In light of the documents, you must make a claim and/or demonstrate a line of reasoning that responds to the prompt. Avoid statements that are vague or general (“The Vietnam War was very significant”) and make a claim that responds to the prompt, uses both the documents and your historical knowledge, and sets up the rest of your essay (“The Vietnam War impacted Americans’ perceptions of the U.S. role in international politics, the power of the federal government, and the status of young people, influencing legal and social changes in American society”).
  • Be sure your thesis or overall plan incorporates a complex understanding . You need to demonstrate that you have more than just a basic understanding of the content, so your thesis or overall essay should address complexity in the historical development—perhaps by including multiple variables, by considering both causes and effects, or by making an insightful connection to another time period. See below for a complete list of ways to demonstrate complex understanding.
  • Make a note about how you will provide context for the topic of the prompt. This may fit well in the introduction or first body paragraph.
  • List the documents you will use as evidence —remember that you must use six or seven to earn the maximum number of points for using the documents.
  • Consider whether the paragraph is a good place to provide additional evidence —you must include one additional historical example.
  • Think about when it would be beneficial to explain sourcing , or how a document’s context or situation is relevant to the argument—you must do so for three documents.
  • Finally, review your plan and check off each requirement in your test booklet to ensure you addressed all six.

During Step 3: Action! Write Your Response

  • Nothing is more important in the first paragraph than the clear statement of an analytical thesis. The reader is most interested in seeing a strong thesis as soon as possible.
  • Your thesis can be more than just one sentence. With the compound questions often asked by the DBQ, two sentences might be needed to complete the idea.
  • Refer to the authors of the documents, not just the document numbers.
  • A good idea is to write a concluding paragraph that might extend your original thesis. Think of a way to restate your thesis, adding information from your analysis of the documents.

During Step 4: Proofread

  • Skim for any glaring errors and, if you have time, check again to make sure your response meets each of the DBQ requirements.

AP Expert Note

Be prepared to demonstrate complex understanding The AP exam asks you to analyze sources and develop arguments in a sophisticated way. Demonstrating your complex understanding of the topic at hand is crucial to your success, and here are some ways you can do so.

  • Analyze multiple variables
  • Employ a complex historical reasoning skill by explaining both similarities and differences, both continuity and change, both causes and effects, or multiple causes
  • Explain relevant connections to other regions or other time periods
  • Corroborate perspectives across multiple course themes (such as environment, cultural developments, governance, economic systems, social organization, and technology)
  • Qualify an argument using other evidence or views

You might also like

APUSH Period 9 Notes

Call 1-800-KAP-TEST or email [email protected]

Prep for an Exam

MCAT Test Prep

LSAT Test Prep

GRE Test Prep

GMAT Test Prep

SAT Test Prep

ACT Test Prep

DAT Test Prep

NCLEX Test Prep

USMLE Test Prep

Courses by Location

NCLEX Locations

GRE Locations

SAT Locations

LSAT Locations

MCAT Locations

GMAT Locations

Useful Links

Kaplan Test Prep Contact Us Partner Solutions Work for Kaplan Terms and Conditions Privacy Policy CA Privacy Policy Trademark Directory

Driver's Ed: Practice Questions

The Magoosh logo is the word Magoosh spelled with each letter o replaced with a check mark in a circle.

APUSH Document Based Questions and Responses: A Study Guide

Document Based Questions tend to freak students out the most on the APUSH exam. This is understandable; not only do you have to read documents, you have to write a coherent essay about them.

However, we’ve got you covered here at Magoosh. For a step-by-step breakdown of what you need to do to write a Document Based Question (DBQ), check out my other blog posts on 3 Steps to a DBQ Essay that Works and How to Write a DBQ Essay . This post, though, will focus on what other students have done to make their Document Based Question essays successful.

I will take you through one DBQ on a prior APUSH exam and give you the ins and outs, and the dos and don’ts. At the end, I will provide a link to a DBQ essay for you to practice some of the things that successful test-takers have done. There, you can compare your essay to the scoring notes provided by College Board. In fact, everything I present on this post will be provided by College Board – you can (and should!) check out their website for more tips.

Sound good? Let’s go!

Document Based Question #1

This is taken from the 2016 APUSH Exam . The DBQ for this section asks you to do the following:

Explain the causes of the rise of a women’s rights movement in the period 1940–1975.

You will have 55 minutes to answer that question. The College Board suggests 15 minutes for reading and 40 minutes for writing, although if you are a fast and careful reader, you can start writing before your 15-minute reading period is done.

I won’t post all of the documents that you have to reference (there are 7 after all!), but the following two documents are representative of the types of documents you will encounter on a DBQ.

how to write a thesis statement for apush dbq

As you can see, there’s a mix of photographs, advertisements, and text that you will be expected to incorporate into your essay.

Still with me? Good. Next, let’s look into what an essay should have in it.

Scoring Notes for Document Based Questions

The following (including descriptions) comes straight from the APUSH scoring notes. I’ll break down parts of it later to make sure that you understand what they want to see.

Your DBQ essay should have the following (for a maximum of 7 points):

  • Thesis: Present a thesis that makes a historically defensible claim and responds to all parts of the question. The thesis must consist of one or more sentences located in one place, either in the introduction or the conclusion.
  • Argument Development: Develop and support a cohesive argument that recognizes and accounts for historical complexity by explicitly illustrating relationships among historical evidence such as contradiction, corroboration, and/or qualification.
  • Use of the Documents: Utilize the content of at least six of the documents to support the stated thesis or a relevant argument.
  • Sourcing the Documents: Explain the significance of the author’s point of view, author’s purpose, historical context, and/or audience for at least four documents.
  • Contextualization: Situate the argument by explaining the broader historical events, developments, or processes immediately relevant to the question.
  • Outside Evidence: Provide an example or additional piece of specific evidence beyond those found in the documents to support or qualify the argument.
  • A development in a different historical period, situation, era, or geographical area.
  • A course theme and/or approach to history that is not the focus of the essay (such as political, economic, social, cultural, or intellectual history).

Yes, it’s a lot. But students have done it before, and so can you! Just because of the limits of space, I am only going to show you what to do – and what not to do – on the parts where I have seen students struggle most: thesis and synthesis.

Thesis Statements for Document Based Questions

Your thesis statement is the bread and butter of any essay you write for the APUSH exam. As I have stated before in previous posts, you should spend the most time on your thesis because a strong thesis will guide the rest of your essay.

But what makes a thesis strong? As mysterious as that question may seem, it is relatively straightforward:

A strong thesis directly answers the question being asked by referencing specific times, movements, or ideas.

It’s that simple! Well, it’s sort of simple. Developing a strong thesis is hard work, but let’s begin at the beginning. Here’s the question being asked: Explain the causes of the rise of a women’s rights movement in the period 1940–1975.

Notice that the question asks for causes, meaning MORE than one. Also, notice that the question gives you a defined time period to work with. Therefore, your thesis shouldn’t deal with any events, ideas, or people outside of that time period.

Let’s look at two student examples.

Example Thesis #1:

The women’s rights movement arose as a result of women’s experiences with inequality at work and the influence of other rights movements.

Does the student directly answer the question being asked? Yes. According to the student, the women’s rights movement was caused by the experiences of women dealing with inequality at work and the influence of other rights movements in the same time.

Is the student being specific? Yes. I know that this student will be organizing their essay in two big chunks: inequality in the workforce and civil rights movements.

Notice that this student didn’t give the longest answer possible, and the response was not necessarily the most eloquent, but that student still got a point for their thesis.

Example Thesis #2:

The woman’s rights movement was the product of unfair treatment in economics, politics, and society.

Does the student directly answer the question being asked? Well, yes, but I am unclear what “economics, politics, and society” means.

Is the student being specific? Not at all. There could be thousands of things that go under economics, politics, and society – and many things could be considered “unfair” – so I have no idea what the student will be arguing in this DBQ.

Thesis Statement Dos and Don’ts

  • Directly answer the question being asked.
  • Be specific.
  • Write a thesis statement like the first example.

Don’t:

  • Answer the question in a confusing way or answer some other question you think the test SHOULD be asking.
  • Be general.

Synthesis in Document Based Questions

This is a newer component of the DBQ. You need to demonstrate your understanding of history by being able to go beyond the documents they provide you and make connections between different parts of history. This does NOT mean that you need to spend all of your time racking your brain for more evidence. However, it does mean that you should have a solid understanding of US History and can extend your argument to other time periods or themes. Let’s look at some student examples to explain what I mean.

Student Example #1:

The conditions that helped cause the rise of the women’s rights movement in the 20th century were similar to those that helped cause the rise of a movement for greater women’s rights in the 1840s. In both periods, calls for greater rights for African Americans led women to demand more of a voice in social and political reforms.

This student explains that the conditions for women’s rights movements were similar in two different time periods, extending the argument beyond this one moment in US history.

Student Example #2:

A development in a different historical period was when Alice Paul went on hunger strikes and protests in from of the White House to gain attention on passing an amendment that would give women their rights. Paul’s fight for women’s rights started with trying to get equal voting rights for women. This links to how in 1940-1975 women were fighting for equal rights in wages and other important rights.

This student makes a connection to another time period by arguing that the fight for equal rights did not begin in 1940; instead, women had been active for some time in US history to achieve equal rights.

Student Example #3:

The Seneca Falls convention also served to help inspire women around the world to gain equal rights. The speech given clearly stated the way things were being conducted was unconstitutional and women should not be socially inferior to men.

Unlike the first two student responses, this third response does NOT connect back to the time period in the question. I am unclear from this student response whether the connection is gaining equal rights for women, the persistence of inequality, or the changing interpretations of the Constitution. This student did not receive a point for the synthesis criteria.

Synthesis Dos and Don’ts

  • Connect back to the time period of the question.
  • State something that you feel is an “obvious” connection, but never make a connection yourself. You should be doing that work for your reader.

Document Based Question #2: Your Turn!

Although I haven’t outlined every single component of the DBQ, you should look at the two blog posts I linked to at the beginning of this article for more references.

But now it’s time for you to dive in! You will only get better by practicing.

You should practice with the 2015 Document Based Question 1 . In that document, you will have access to the questions and sample student responses.

Good luck, and let me know how it goes!

Allena Berry

Allena Berry loves history; that should be known upfront. She loves it so much that she not only taught high school history and psychology after receiving her Master’s degree at Stanford University, she is now studying how students learn history at Northwestern. That being said, she does not have a favorite historical time period (so don’t bother asking). In addition to history, she enjoys writing, practicing yoga, and scouring Craigslist for her next DIY project or midcentury modern piece of furniture.

View all posts

More from Magoosh

APUSH Short Answer Questions and Responses: A Study Guide

Leave a Reply Cancel reply

Your email address will not be published. Required fields are marked *

logo-type-white

AP® US History

The ultimate guide to the 2015 ap® us history dbq.

  • The Albert Team
  • Last Updated On: March 1, 2022

The Ultimate Guide to the 2015 AP® US History DBQ

The AP® US History Document-Based Question (DBQ) can be intimidating at first. With some practice and careful studying, though, DBQs can be a lot of fun.

One of the most useful things you can do to ensure a high score on the DBQ is to look over past DBQ questions. Previous years’ questions give you an opportunity to practice your historical thinking and writing skills. Also, the CollegeBoard releases information about how they graded each one of these prompts, so you can get a sense of what you should emphasize in your writing and how you would have done on that exam.

To help you study for this year’s AP® U.S. History DBQ, we will spend this post walking through the 2015 U.S. History DBQ. You will learn what the 2015 test-takers did right and wrong, as well as what you need to do to get full credit when you take the exam. By the end of the post, you will have a solid grasp of what graders were looking for in the 2015 exam as well as what you need to do more generally to get full credit on the DBQ section of the test.

Before we dive deep into the 2015 DBQ prompt, however, let us briefly review what the U.S. History DBQ is and how the CollegeBoard scores it.

Format of the AP® US History DBQ

Be aware that resources from before the 2014-2015 school year detail an old AP® U.S. History exam format. The CollegeBoard now uses an exam format with different standards. Here, we focus on the current form.

The DBQ asks you to make an argument based on a series of included historical documents as well as your knowledge of United States historical context (see here ). You have 55 minutes to write your response. The CollegeBoard suggests that you spend 15 of those minutes reading the documents and the remaining 40 minutes writing your essay.

You can earn a maximum of seven points for the DBQ question. The points are split up into the following seven categories (see here ), with one point for each category:

  • Argument Development
  • Use of the Documents
  • Sourcing the Documents
  • Contextualization
  • Outside Evidence

Each point in the rubric is earned independently, meaning if you miss one point, you will not necessarily get marked down on other points as well (see here ).

However, keep in mind that you need to show unique evidence that you address each point in the rubric. So, if you receive a point for sourcing documents in a particular sentence, you cannot also receive points for your thesis in the same sentence.

We will go through the specifics of how you can get each of these seven points in a bit, but first, let us consider the DBQ prompt for the 2015 AP® U.S. History Exam, so you can see examples of how real DBQ responses were scored based on these seven criteria.

The 2015 AP® U.S. History DBQ Prompt

In 2015, the DBQ prompt was as follows (see here ):

Explain the reasons why a new conservatism rose to prominence in the United States between 1960 and 1989.

Along with the prompt were six included documents, each meant to be analyzed and synthesized for you to build an argument based on the prompt. Here we will just summarize each one so you can follow along, but you are encouraged to look at the full documents by clicking on their links below.

  • A passage from Barry Goldwater’s 1960 book The Conscience of a Conservative (see here ), expressing concern for what he terms “Big Government”.
  • A section of Milton Freedman’s 1962 book Capitalism and Freedom (see here) , arguing that free markets better enable social progress than government interventions.
  • A letter to Nelson Rockefeller (governor of New York) in 1971 from a constituent concerned by what she perceives as policies that favor drug addicts and people on welfare (see here ).
  • An excerpt from Listen America , by Jerry Falwell (television evangelist) that argues Americans have lost their biblical foundations, resulting in widespread immorality (see here ).
  • The 1980 Republican Party Platform emphasizing their view that the Carter Administration was diplomatically weak (see here ).
  • An excerpt from Teddi Holt (1984), a homemaker and president of Mothers On the March, who argues for traditional male and female gender roles based on biblical principles (see here ).

How to Get All 7 Points in the AP® US History DBQ

how to write a thesis statement for apush dbq

1. Write a Strong Thesis Statement

What is the main point you are trying to get across in the essay? AP® graders want you to clearly write a thesis statement in either your introduction or conclusion. A good thesis statement should make a historically defensible claim and encapsulate the entire argument for your essay in a sentence or two (see here ). A good thesis should also respond to all parts of the DBQ prompt. If you write a thesis that succeeds at checking all these boxes, graders will award you your first point. Let us examine what a good thesis statement would look like for the 2015 DBQ and how to avoid writing a bad one.

A 2015 student, for instance, posed the following argument (see here ):

“Many claim that the new conservatism rose to prominence in the U.S. between 1960 and 1989 because of the instability of the economy. However, three more important causes were the left activist influence on politics, the break-up of the traditional family life, and the effects of the counter-culture within society. Therefore, the rise of the new conservatism resulted from the desire for a return to structure and order.”

This thesis is good because it makes a historically defensible claim that explains the rise of new conservatism. Specifically, the claim stresses political and social changes as drivers for the movement from 1960 through 1989. As such, it responds to all parts of the prompt as well.

The specifics of your argument depend on what you find most interesting about the question, what you know most about the surrounding historical context, and the included document’s themes. For instance, another good thesis from a student who took the 2015 exam concentrated on the economic and foreign policy reasons for the rise of new conservatism (see here ):

“The most important factors that contributed to the birth of the new conservative movement were a desire for more reliance on free-market capitalism, a society oriented toward traditional morals and values, and a government that was strong on both foreign and domestic policy.”

Both theses go beyond rewriting the prompt and specifically address the characteristics of historical change that the students plan to discuss in the remainder of their response.

If your thesis makes a historically defensible claim, responds to all parts of the question, and occurs in the introduction or conclusion, the graders will give you one point for successfully writing a thesis statement (see here ).

A bad thesis statement, on the other hand, may not include one or all of the things that make a good thesis. For instance, such a thesis may be vague and not contain a historically defensible claim, like this one from a 2015 test-taker (see here ):

“The period from the 1960’s to the 1980’s saw the rise of a political movement known as the new conservatism. The movement grew due to social, economic, and political tides which existed during the Cold War era.”

While the student mentioned the topic, they just restated the prompt, with only vague indications of what they plan to talk about next. To make a historically defensible claim, they need to be more specific about what they mean by “social, economic, and political tides” for instance.

On the other hand, a thesis might successfully make a historically defensible claim, but fail on other counts. Take this statement from another 2015 test taker’s DBQ response (see here ):

“Conservatives wanted smaller government, lower taxes, and stronger foreign policy.”

While the statement is a historically defensible claim, it does not address the prompt. The prompt specifically asks for reasons why new conservatism rose from 1960-1989—not just for characteristics of conservatism. To get credit for having a thesis, you need to address every aspect of the prompt fully. Read the prompt carefully! One of the most common problems on the 2015 exam was that students misread the prompt and focused on a small part of it rather than explicitly answering the question (see here ).

2. Develop the Thesis into a Cohesive Argument

To earn a point for argument development, graders want you to avoid simply listing facts in support of your thesis. The CollegeBoard wants you to explicitly illustrate the relationship between your thesis and each piece of evidence you cite (see here ). Instead of only listing a string of facts and documents from memory, they want you to talk about how each piece of evidence either contradicts, corroborates, or qualifies your main thesis. Let us take a look at a couple of good and bad ways of developing arguments in the 2015 DBQ.

For instance, one test-taker in 2015 effectively used a contradiction between the substance of two included documents to flesh out the argument for their thesis (see here ):

“The main reasons for a rise of conservatism, according to conservatives such as Friedman, were the need for greater individual freedom. However, some religious conservatives did not agree. Falwell (Document 4) and the Moral Majority saw the problem as immoral behaviors and selfishness encouraged by the media. They called for less individual freedom and more respect for authority.”

Another test-taker earned a point by corroborating their thesis with historical evidence (see here ):

“The conservative movement arose due to a backlash against the prevalence of liberals and the counter-culture in the period. Holt (Document 6) attacks the rise of feminism; similarly, Jerry Falwell argues that young people have been taught moral ambiguity and introduced to a “drug culture” (Document 4). Other conservatives complained about anti-war and civil rights protests in the 1960’s.”

A third way to relate historical evidence to your overall thesis is to provide qualification. For instance, one student qualified the new conservative movement by emphasizing their advocacy for military spending (see here ):

“Although the biggest reason for the rise of the conservative movement was public opposition to the expansion of the federal government, conservatives made an exception in this position by advocating more spending on the U.S. military in fighting the Soviet Union.”

All three of these excerpts illustrate that the test-takers developed their thesis into a cohesive argument and would have earned one point for the argument development section of the rubric.

On the other hand, graders will not award you a point if you string together facts without explicitly relating them back to your thesis. For instance, bad argument development might just list the facts of the different documents without using them to contradict, corroborate, or qualify the thesis. To illustrate, let us rework one of the good samples above:

“Friedman favored greater individual freedom (Document 2). Falwell (Document 4) and the Moral Majority argued that the media encouraged immoral behavior and selfishness. They called for people to respect authority.”

Note that the facts used in these sentences are the same as in the example of good development of argument through contradiction. However, while the facts are relevant to the question, the writer does not explicitly address how they come to bear on the thesis. As a result, we are left not knowing how the writer intends to mobilize the documents to make their point. This point is subtle but important: make sure you are making an explicit argument with the historical evidence. Otherwise, you will not receive a point for the argument development section of the DBQ rubric.

3. Use 6 Documents from the Included Documents

how to write a thesis statement for apush dbq

To get the point for the “Use of the Documents” section of the DBQ rubric, you will need to use the at least six of the documents to support your thesis. It is not enough to simply quote or paraphrase a document for it to count, though. You must explicitly connect it to your thesis (i.e. in the same way as the Argument Development point). Let us take a look at good and bad examples of document references from the 2015 DBQ.

For instance, for one of their six document references, one 2015 test-taker wrote (see here ):

“Beginning in 1960, Republican candidates began to stress the liberal encroachment of all our civil liberties. Goldwater, a Republican senator, acknowledged that Democrats like Roosevelt had defied the constitution that Americans held dear.”

In this case, the writer explicitly uses the information from Document 1 to support the notion that the new conservatism arose as a result of perceived liberal departures from the Constitution. If the writer successfully applies at least six documents to the thesis in this way, the writer will receive one point for “Use of the Documents”.

Contrast the above excerpt with this 2015 test taker’s work (see here ):

“Teddi Holt was a homemaker and a member of the growing organization ERA (Equal Rights Association) for women. She held the point of view that women needed to speak out against loss of values and discrimination.”

While the test-taker successfully mentioned Document 6, they misread the point of the document (with an emphasis on preserving traditional gender roles). Furthermore, the writer does not say how this point connects to a larger discussion of how the desired return to traditional values fueled new conservatism.

To get the point for the “Use of the Documents” section of the AP® U.S. History DBQ, be sure to include your documents only in service of your larger thesis. For each document you add to your response, you should ask yourself the question: “How does this document relate to my thesis?” Be sure to explicitly state your answer to the question whenever you mention a document. Otherwise, you will miss out on one of the 7 DBQ points.

4. Explain the Significance of 4 Documents’ Social and Historical Context

To get the point for the “Sourcing the Documents” section of the rubric, you must account for the significance of four documents’ social and historical context. For specific examples of test-takers who successfully explained the significance of DBQ documents, consider the following “good” examples.

For instance, you may mention the significance of the document author’s point of view in forming the argument they present in the document. One test-taker wrote (see here ):

“Milton Friedman refuted the idea that welfare programs help solve the problems they are aimed at (Doc 2). As a conservative economist, Friedman makes a case for why capitalism and markets are better than welfare spending.”

The student mentions Friedman’s point of view as a conservative economist as a means of qualifying what he said about welfare programs. After all, his statements could have vastly different social implications if a liberal economist, or a director of a government welfare agency wrote them.

You might also explain the significant of an author’s purpose in creating the document. For instance, another test-taker wrote (see here ):

“Jerry Falwell (Document 4) attacks the moral decay and de-emphasis of American exceptionalism as the leader of the ‘Moral Majority,’ for the purpose of inspiring political action to correct the potentially corrupting influences of the media on children.”

The test-taker explicitly addresses how Falwell’s purpose in writing Document 4 plays a role in interpreting the document. Further treatment of the source may deal with how the significance of purpose plays into the document’s usefulness as a source of historical information.

Another approach to take is to explain how the document fits into larger historical trends and what significance these larger trends may have on the interpretation of the document. For instance, one test-taker addressed the context surrounding the 1980 Republican platform statement (Document 5) (see here ):

“A major issue of Carter’s presidency was the Iran hostage crisis, where Iranian students stormed the U.S. embassy and took hostages. Carter’s lack of response made the U.S. look diplomatically and militarily weak. In the 1980 Republican platform (Doc. 5), the authors allude to the humiliation of the hostage crisis and vow never to show that kind of weakness.”

The significance here of the document’s surrounding historical context lies in the national emotions surrounding Jimmy Carter’s presidency and how the Republican Party tapped into those emotions in forming their 1980 platform.

The intended audience might also be a significant factor in the authorship of a document. Another student wrote (see here ):

“Writing as the leader of the movement ‘Mothers on the March,’ Teddi Holt (Document 6) creates a sense of community with other ‘mothers’ in her writing by asking them to stand up for their homes and their children.”

The test-taker takes into account Holt’s audience, making the particular language she uses in her document more understandable in a social and historical context.

If you use any of the preceding approaches to demonstrate the significance of the social and historical context of four of your documents, you will receive a point for the “Sourcing the Documents” section of the rubric.

A common mistake that test-takers make is to mention the social and historical context of a source, but not address the significance of that context for the production of the document. You must address the significance of the social and historical context for four documents to get the “Sourcing the Documents” point on the DBQ rubric.

For instance, while this student mentions the author’s point of view, they do not explain its significance (see here ):

“The Republican Party Platform (Document 5) obviously expresses the conservative view of one political party in that year’s presidential election.”

Merely stating the point of view of an author is not enough to get you a point. You must address the role that point of view played in generating the document at hand.

Similarly, another student explicitly mentions the purpose of a document (see here ):

“The purpose of the letter to Rockefeller (Document 3) is describing the crime and poverty faced by law-abiding citizens of New York in 1971.”

However, the student makes no effort to analyze why this purpose played a role in driving the citizen to draft a letter to Governor Rockefeller and how the citizen’s purpose might play into how we use the document as evidence in support of our thesis.

In the same way, if you only hint at the intended audience without explicitly telling the reader why the audience significantly influences the document’s content, you will not receive credit. Consider this student’s response (see here):

“Falwell (Document 4) is a television evangelist who tells his audience that television introduces children to drugs and permissive values.”

While the test-taker mentions an audience, they do not discuss the role this audience played in the construction of Falwell’s message. In every instance, be sure to explicitly state why the social or historical context you are describing is important for the generation of the document.

One thing to keep in mind, however, is that describing a social or historical context and its significance is not enough. To receive credit, you must describe the correct historical context for the document. For instance, this student incorrectly attributes Barry Goldwater’s Document 1 statements to an earlier period (see here ):

“The context of Document 1 is due to Goldwater’s attempt to use this as a platform for his losing presidential campaign against Johnson’s Great Society.”

In summary, to ensure that you get credit for this section of the DBQ be sure to describe the correct context for four of the documents (whether that be the author’s point of view, purpose, historical context, or audience) as well as explicitly state the significance of each context. It would be a shame to get marked down for something you implicitly know but did not say explicitly!

5. Contextualize Your Thesis in Light of Broader Trends in United States History

To get the “Contextualization” point, you will need to state how your thesis fits into the larger U.S. historical context. Specifically, you should contextualize your position based on your understanding of U.S. History outside of the included documents. Let us take a look at what good and bad examples of contextualizing might be for the 2015 DBQ.

Some relevant trends that the CollegeBoard looked for when they graded the “Contextualization” point in the 2015 DBQ included the following (see the full list here ):

  • Reactions against the perceived permissiveness of the 1960s and 1970s, including the counterculture, anti-war protest movement, feminism, and the sexual revolution
  • The Cold War and the ongoing thread of anticommunism in the post- World War II United States
  • Reactions against the perceived excesses of government interventions in the economy and society, such as New Deal or social welfare programs, or the Johnson administration’s Great Society agenda
  • Concerns about United States economic stagnation, combined with belief in free-market solutions
  • Concerns about increased crime and the perceived need for greater law and order

Notice that each one of these are broad-strokes but correct statements about general patterns in U.S. History. If you contextualize your thesis in one of these ways, you will receive a point for “Contextualization” in the rubric.

It can be tough to practice the “Contextualization” point since you will not know the prompt ahead of time. However, the CollegeBoard will not ask any questions that focus exclusively on events before 1607 or after 1980 (see here ). So, if you study the remaining seven major periods the AP® U.S. History Exam focuses on, you should be able to write a sentence or two about the broad trends from any one of these periods. For the DBQ, it will then be easier for you to address how your thesis fits into these broad trends you’ve studied.

You will not receive a point for this section if you discuss a historical context outside the question’s time period. So, for instance, one student wrote (see here ):

“The new conservatism began to rise during the Eisenhower Administration and McCarthy’s attacks on Communists in government.”

While the statement may be tangentially relevant, it does not contextualize the specific rise of new conservatism from 1960-1989—the focus of the question.

Another common way to lose a point for this section was to talk about the historical trends of the period, but not to explicitly tie them into the focus of the question: the rise of conservatism. For example, another student argued that (see here ):

“The rise of the Cold War led many Americans to become more patriotic and defensive of American values and spend more on the military. Conservatives rose to object to perceived attacks on capitalism by liberals and leftists in the 1960’s and 1970’s.”

While this is a descriptive statement of the period, the test-taker did not tie these broader trends into why a new conservatism arose. They might have mentioned, for instance, what about the Cold War inspired particularly conservative American values or how liberals and leftists were perceived as attacking capitalism. Thus, even though the test-taker correctly identified trends, they did not receive a point for “Contextualization.”

6. Provide a Piece of Evidence Beyond the Documents

how to write a thesis statement for apush dbq

Be sure to include an additional piece of evidence to support your thesis beyond the documents. To to be awarded the “Outside Evidence” point on the rubric, graders want you to discuss the material you have studied about the period. Let us take a look at what good and bad examples of additional evidence might be for the 2015 DBQ.

For instance, some of the terms and events the CollegeBoard looked for in the 2015 DBQ were as follows (see here for the full list):

  • Antiwar protests
  • Assassinations of John F. Kennedy, Martin Luther King Jr., Robert F. Kennedy, and Malcolm X
  • Black Power movement
  • Civil Rights Act (1964)
  • Debates over nuclear weapons
  • Deregulation of industry
  • Environmental Protection Agency (EPA)
  • Equal Rights Amendment (ERA)
  • Reagan’s description of the Soviet Union as an “evil empire.”

To get credit for your knowledge of these events, you should explicitly state how your outside knowledge supports your overall thesis. For instance, one student wrote (see here ):

“The legalization of interracial marriage by the Supreme Court and the increase in numbers of openly gay people, as well as the association of the youth counterculture with experimentation with drugs and sex, provided conservatives like Falwell with evidence of changing social morals.”

The student ties their knowledge of outside events in with their argument for how new conservative attitudes developed during the period. The student even transitions into talking about Jerry Falwell, in a transition back towards their discussion of Falwell’s Document 4 excerpt.

On the other hand, if you do not mention any evidence outside of the included documents, you will not receive a point for “Outside Evidence.”

Additionally, outside evidence that is not related to the time period of the question or not specifically connected to the question of conservatism will not pass muster. One student wrote, for example (see here ):

“The actions of Presidents Truman and Eisenhower created a sense of social security that Americans had come to rely upon.”

Truman and Eisenhower’s presidencies occurred before the question’s time period, and their policies are thus not relevant for answering the question. Using external evidence in this way would not result in a point for the section.

7. Show how Your Argument Corresponds to Larger Historical Themes or Developments in other Historical Periods

The CollegeBoard devotes one “Synthesis” point in the US History DBQ rubric to how well you extend your argument and show how it corresponds to larger historical themes or developments in other historical periods. Let us take a look at a couple of good and bad ways test-takers extended their arguments in the 2015 exam.

One way of extending your argument to earn the “Synthesis” point is to connect the argument to a development in a different historical period, situation, era, or geographical area (see here ). For instance, this student compares the rise of the new conservatism to a similar process in the Second Great Awakening in the early 19 th century (see here ):

“The rise of a new outlook in response to the troubles of society is similar to what happened during the Second Great Awakening. During that time, many religious people in America were unsatisfied with the way their society had developed. Thus, in both periods, societal developments reinvigorated religious thinking and helped the U.S. return to past ideals.”

Other successful syntheses for the 2005 DBQ prompt might include comparing the rise of the new conservatism to earlier coalitions that formed concerning the New Deal, or linking the rise of the new conservatism in this period to the later emergence of the Tea Party (for more ideas, see here ).

Another good way to extend your argument is to connect your thesis to broader themes outside the narrow bounds of the question itself. For instance, one test-taker related the rise of new conservatism in terms forming new types of American identities (see here ):

“Although political and social factors were important to the rise of conservatism, the movement can be seen as creating a new form of identity similar to that of ethnic groups that advocated for rights in the 1960’s.”

The CollegeBoard emphasizes that thematic connections must go beyond just mentioning a phrase like “identity” (see here ). Instead, thematic connections must make a substantive claim about how the theme could have led to rising conservatism. Note that the test-taker above clearly states how identity groups like ethnic groups in the 1960’s advocated for rights and constructed their identities in a similar way that new conservative identities emerged from 1960-1989.

If you extend your argument in either of these two ways, you will earn a point for the “Synthesis” component of the DBQ rubric.

You will not earn a “Synthesis” point, however, if you do not accurately connect your argument to a historical development or theme. For instance, this 2015 test-taker mentioned another historical development, but did not explain the way that the events from the two periods correspond (see here ):

“The rise of conservatism during this period mirrored that which arose during the changing social conditions and rapid influx of immigrants during the Gilded Age.”

If you choose to emphasize a thematic connection, note that it must be outside the main focus of the question. Otherwise, the thematic connection is not a comparison or a novel synthesis! It is simply supporting evidence, identical with the main point of the essay. For instance, this test-taker argued (see here ):

“Although the new conservatism mostly arose due to a reaction against liberal social programs, it also benefited from public disappointment with politicians in general.”

Because the 2015 DBQ prompt is fundamentally about a rising political conservative movement that emerged between 1960 and 1989, this excerpt does not add thematic perspective on the phenomenon beyond what the prompt and the documents give us. Therefore, this test-taker would not receive a “Synthesis” point.

Now that we have walked through the steps to get all seven points on the 2015 DBQ, it is your turn to practice!

It is one thing to read about writing responses. To truly prepare for the AP® U.S. History DBQ, though, you need to practice writing DBQ responses. Now that you know exactly what test-makers are looking for in the 2015 response, why don’t you try your hand at reading the documents and writing your own response to the same prompt ?

Focus on including all 7 points we have talked about here. Try to mimic the good test-takers from 2015 in earning each DBQ point. If you practice taking enough DBQs, it will become automatic for you to write a DBQ essay and it will be a breeze when you take the exam.

Practice makes perfect!

Looking for AP® US History practice?

Kickstart your AP® US History prep with Albert. Start your AP® exam prep today .

Interested in a school license?​

Popular posts.

AP® Physics I score calculator

AP® Score Calculators

Simulate how different MCQ and FRQ scores translate into AP® scores

how to write a thesis statement for apush dbq

AP® Review Guides

The ultimate review guides for AP® subjects to help you plan and structure your prep.

how to write a thesis statement for apush dbq

Core Subject Review Guides

Review the most important topics in Physics and Algebra 1 .

how to write a thesis statement for apush dbq

SAT® Score Calculator

See how scores on each section impacts your overall SAT® score

how to write a thesis statement for apush dbq

ACT® Score Calculator

See how scores on each section impacts your overall ACT® score

how to write a thesis statement for apush dbq

Grammar Review Hub

Comprehensive review of grammar skills

how to write a thesis statement for apush dbq

AP® Posters

Download updated posters summarizing the main topics and structure for each AP® exam.

United States History Writing Study Skills

Frank Warren, a history professor at Queens College and a former Chief Faculty Consultant for AP U.S. History, offers the following suggestions for writing a good response to a document-based question (DBQ) or free-response essay question.

Write More Often

AP students need to write, and to write often. This practice is an excellent way to develop the skill of casting a thesis statement and marshalling evidence in support of a valid generalization.

Define Your Terms Where Necessary

Look especially at terms like liberal or conservative, radical or progressive. Be prepared to define other central terms, such as major change, that may appear to be obvious but can be ambiguous.

Start with a Clearly Stated Thesis

Some good essay writers begin with a thesis statement, back it up with supporting evidence from documents and outside knowledge, and, if time permits, restate the thesis at the end. Other writers analyze the material and build up logically to their thesis statement. On an AP Exam, you should use whichever method you feel most comfortable with. In any case, exam day is probably not a good time to experiment with a new, unfamiliar method of writing.

Organize Your Response Carefully

In addition to having a strong thesis, it is a good idea to have a guiding organizational principle—a stated agenda for making your point. Try to integrate your outside information into your response. Your exam shouldn’t read as if you threw in a few tidbits of outside information at the end.

Make Sure Thesis Matches Assessment and Knowledge

Many good essay writers demonstrate a sense of complexity in the documents, showing that most of the evidence may point in one direction but that part of the evidence points in a different direction. It is better, however, to support a clear, simple thesis than to create artificially a complexity that you can't support using the documents or outside knowledge. Almost every essay—including the DBQ—is designed to allow the student to agree or disagree with the statement. Your ultimate goal should be to present a well-argued and well-supported thesis, not merely to give the people scoring the essay what you think they want.

Build an Argument

The best essays—in terms of arguing their case—are those that marshal the positive arguments in favor of their position but that also refute or answer possible rival theses. Even if you think a statement is completely true, it is better to confront and negate the evidence that seems to refute it than to ignore the counterevidence completely.

Integrate the Documents and Your Analysis

Don’t merely explain what is stated in the documents. Rather, use the documents as part of an integrated essay in support of your thesis.

Don’t Quote Large Portions of the Documents

The readers of the essays are already familiar with the documents. You can quote a short passage or two if necessary, to make your point, but don’t waste time or space reciting them.

Choose Your Essays Wisely

Select the questions you are best prepared to answer. The questions that invite the easiest generalizations are not always the ones you should answer. As you read through the questions and make your choices, ask yourself for which of the questions are you best prepared to support your thesis.

PrepScholar

Choose Your Test

  • Search Blogs By Category
  • College Admissions
  • AP and IB Exams
  • GPA and Coursework

Where to Find the Best DBQ Examples

author image

Advanced Placement (AP)

feature-google-search-cc0-pixabay

One of the best ways to prepare for the DBQ (the "document-based question" on the AP European History, AP US History, and AP World History exams) is to look over sample questions and example essays. Doing this will help you to get a sense of what makes a good (and what makes a bad) DBQ response.

That said, not all DBQ essay examples are created equal. We'll briefly cover what makes a good DBQ example and then provide a list of example essays by course. Lastly, we'll give you some helpful tips on how to best use sample essays in your own preparation process.

What's a Good DBQ Example?

Without a doubt, the best sample resources come from the College Board . This is because they are the ones who design and administer the AP exams . This means the following:

Any DBQ essay example that the College Board provides will include a real DBQ prompt

All samples are real student responses from previous years , so you know they were written under the same conditions you'll have when you write your DBQ—in other words, they're authentic!

They not only have scores but also explanations of each essay's score , in accordance with the rubric

Each prompt includes several sample essays with a variety of scores

Some DBQ examples outside those available from the College Board might be worth looking at, particularly if they highlight how a particular essay could be improved. In general, though, a superior example will do the following:

Include the prompt and documents: It will be much easier for you to see how the information from the documents is integrated into the essay if you can actually look at the documents themselves!

Have a score: Seems simple, but you'd be surprised how many DBQ examples out there in the uncharted internet don't have one. Without a real, official score, it's hard to gauge how trustworthy a sample actually is.

With that in mind, I have compiled lists, organized by exam, of high-quality example DBQs below.

body-pencil-notes-cc0-pixabay

Don't spend all your study time on false starts with your practice DBQs. 

Every DBQ Example Essay You Could Ever Need, by Exam

Here are your example essays! We'll start with AP US History, then move to AP European History, and finally wrap up with AP World History.

AP US History: Official College Board Examples

The APUSH test was redesigned in 2015 and again in 2018, so right now there are eight  official College Board sets of sample essays you can use in your studies . Make sure to give yourself a 15-minute reading period and 45 minutes to write your answer. In addition, don't forget to use the current scoring guidelines when grading your own practice responses.

  • 2023 Free-Response Questions | Scoring Guidelines 2023
  • 2022 Free-Response Questions | Sample DBQ Responses 2022
  • 2021 Free-Response Questions | Sample DBQ Responses 2021
  • 2019 Free-Response Questions | Sample DBQ Responses 2019
  • 2018 Free-Response Questions | Sample DBQ Responses 2018
  • 2017 Free-Response Questions | Sample DBQ Responses 2017
  • 2016 Free-Response Questions | Sample DBQ Responses 2016
  • 2015 Free-Response Questions | Sample DBQ Responses 2015

If you want additional sample question sets, you can look at older College Board US History DBQ example response sets . To look at these, click "Free-Response Questions" for a given year. For the corresponding DBQ examples and scoring guidelines, click "Sample Responses Q1."

Note that these examples use the old rubric (which is integrated into the Scoring Guidelines for a given free-response section). General comments on the quality of the essay, outside information, and document analysis still apply, but the score is on a 9-point scale instead of the current 7-point scale, and some of the particulars will be different. Older DBQs had up to 12 documents, while the current format has seven documents.

If you do look at older DBQ examples, I recommend using the current rubric to re-grade the essays in the sample according to the 7-point scale. I'll also give more advice on how to use all these samples in your prep later on.

body-eagle-cc0-pixabay

Mr. Bald Eagle is an AP US History DBQ grader in his spare time.

AP European History: Official College Board Examples

Unfortunately, there aren't as many sample resources for the AP Euro DBQ compared to the other AP history tests because 2016 was the first year the AP Euro test was administered in the new format . Since then, more minor changes have been made in terms of time (you now have an hour on the DBQ) and individual parts of the rubric (you can view the current scoring guidelines here ).

This means there are seven sets of official samples graded with the current 7-point rubric:

The rest of the existing available samples were graded in the old 9-point format instead of the 7-point format implemented in 2016.

In the old format, there were 6 "core" points and 3 additional points possible. The old rubric is integrated with the sample responses for each question, but we'll highlight some key differences between the old and current formats :

With the old format, you were given a brief "historical background" section before the documents

There were more documents—up to 12—but the current format has seven

There was an emphasis on "grouping" the documents that is not present in the current rubric

There was also explicit emphasis on correctly interpreting the documents that is not found in the current rubric

While the essential components of the DBQ are still the same between the two test formats, you should definitely refer to the current rubric if you decide to look at any old AP European History samples . You might find it useful to look at old essays and score them in accordance with the current rubric.

Here are the old sample DBQ questions and essays, organized by year:

  • 2014 Free-Response Questions | Sample DBQ Responses 2014
  • 2013 Free-Response Questions | Sample DBQ Responses 2013
  • 2012 Free-Response Questions | Sample DBQ Responses 2012
  • 2011 Free-Response Questions | Sample DBQ Responses 2011

You can get samples in the old format all the way back to 1999 from the College Board . (Click "Free -Response Questions" for the questions and "Sample Response Q1" for the samples.)

body-castle-cc0-pixabay

Consider how you might integrate this castle into the DBQ that is your life.

AP World History: Official College Board Examples

The World History AP exam transitioned to a new format to more closely resemble AP US History and AP European History for the 2017 test. This means that there are six past exams available that use the current DBQ format:

Note that starting with the 2020 exam, AP World History will only cover the years 1200 to the present instead of thousands of years of history. As a result, both the course and exam have been renamed AP World History: Modern (a World History: Ancient course is in the works). What this means for you is that previous DBQs might have to do with time periods you're no longer required to study, so just keep this in mind.

In the old format, there were 7 "core" points and 2 additional points possible. The old rubric is integrated with the sample responses for each question, but we'll highlight some key differences between the old and current formats :

There were more documents—up to 10—but the current format has seven

There was an emphasis on "grouping" the documents on the old rubric that is not present in the current rubric

  • In the old rubric, you needed to identify one additional document that would aid in your analysis; the new rubric does not have this requirement

The essential components of the DBQ are still the same between the two formats, though you should definitely look at the current rubric if you study with any old AP World History questions and samples. You might find it useful to look at the old essays and score them according to the current rubric.

Here are old AP World History questions and DBQ sample responses , organized by year:

body-sphynx-cc0-pixabay

Don't worry, the old format isn't as old as this guy right here.

How Should I Use DBQ Examples to Prepare?

Now that you have all these examples, what should you do with them? In this section, we'll give you some tips on how to use example DBQs in your own AP history prep , including when to start using them and how many you should plan to review.

What Should I Do With These DBQs?

Official sample essay sets are a great way to test how well you understand the rubric. This is why we recommend that you grade a sample set early on in your study process—maybe even before you've written a practice DBQ .

Then, when you compare the scores you gave to the official scores and scoring notes given to the samples, you'll have a better idea of what parts of the rubric you don't really understand . If there are points you are consistently awarding differently than the graders, you’ll know those are skills you'll need to work on.

Keep giving points for the thesis and then finding out the sample didn't get those points? This tells you to work more on your thesis skills. Not giving points for historical context and then finding out the AP grader gave full credit? You need to work on recognizing what constitutes historical context according to the AP.

Check out my tips on building specific rubric-based skills in our guide on how to write a DBQ .

Once you've worked on some of those rubric skills you're weaker in, such as evaluating a good thesis or keeping track of how many documents were used, grade another sample set. This way you can see how your ability to grade the essays like an AP grader improves over time!

Obviously, grading sample exams is a much more difficult process if you're looking at examples in an old format. The old scores as awarded by the College Board will be helpful in establishing a ballpark —a 9 is still going to be a good essay using the current 7-point scale—but there may be some modest differences in grades between the two scales. (For example, maybe that perfect 9 is now more like a 6 out of 7 due to rubric changes.)

For practice grading with old samples, you might want to pull out two copies of the current rubric, recruit a trusted study buddy or academic advisor (or even two study buddies!), and have each of you re-grade the samples .

You can then discuss any major differences in the grades each of you awarded. Having multiple sets of eyes will help you determine whether the scores you're giving are reasonable, since you won’t have an official 7-point College Board score for comparison.

Looking for help studying for your AP exam? Our one-on-one online AP tutoring services can help you prepare for your AP exams. Get matched with a top tutor who got a high score on the exam you're studying for!

How Many Example DBQs Should I Be Using?

The answer to this question depends on your study plans.

If it's six months before the exam and you plan on transforming yourself into a hard diamond of DBQ excellence, you might do practice grading on a sample set every few weeks to a month to check your progress to being able to think like an AP grader. In this case, you would probably use six to nine official sample sets.

If, on the other hand, the exam is in a month and you're just trying to get in some extra skill-polishing, you might do a sample set every week to 10 days . It makes sense to check your skills more often when you have less time to study because you want to be sure that you are focusing your time on the skills that need the most work. For a short time frame, expect to use somewhere in the range of three to four official sample sets.

Either way, you should be integrating your sample essay grading with skills practice and doing some practice DBQ writing of your own .

Toward the end of your study time, you could even integrate DBQ writing practice with sample grading. Read and complete a timed prompt and then grade the sample set for that prompt, including yours! The other essays will help give you a sense of what score your essay might have received that year and any areas you might have overlooked.

There's no one-size-fits-all approach to using sample sets, but in general they are a useful tool for making sure you have a good idea what the DBQ graders will be looking for when you write your own DBQ on test day.

body-ostriches-cc0-pixabay

Hey, where can we find a good DBQ around here?

Closing Thoughts: Example DBQs for AP History Tests

Example DBQ essays are a valuable resource in your arsenal of study strategies for the AP history exams. Grading samples carefully will help you get a sense of your own blind spots so you'll know what skills to focus on in your prep.

That said, sample essays will be most useful when integrated with your own targeted skills prep . Grading 100 sample essays won't help you if you aren't practicing your skills; rather, you'll just keep making the same mistakes over and over again.

Make sure you aren't using sample essays to avoid writing practice DBQs either—you'll want to do at least a couple, even if you only have a month to practice.

And there you have it, folks. With this list of DBQ examples and tips on how to use them, you are all prepared to integrate samples into your study strategy!

body-whats-next-cc0-pixabay

What's Next?

Still not sure what a DBQ is? Check out my explanation of the DBQ to learn the basics.

Want tips on how to really dig in and study for AP history tests? We've got a complete how-to guide on preparing for and writing the DBQ .

If you're still studying for AP World History, check out our top AP World History study guide , or get more practice tests from our complete list .

Want more study material for AP US History? Look into this article on the best notes to use for studying from one of our experts. Also, read our review of the best AP US History textbooks !

Want to improve your SAT score by 160 points or your ACT score by 4 points?   We've written a guide for each test about the top 5 strategies you must be using to have a shot at improving your score. Download them for free now:

Trending Now

How to Get Into Harvard and the Ivy League

How to Get a Perfect 4.0 GPA

How to Write an Amazing College Essay

What Exactly Are Colleges Looking For?

ACT vs. SAT: Which Test Should You Take?

When should you take the SAT or ACT?

Get Your Free

PrepScholar

Find Your Target SAT Score

Free Complete Official SAT Practice Tests

How to Get a Perfect SAT Score, by an Expert Full Scorer

Score 800 on SAT Math

Score 800 on SAT Reading and Writing

How to Improve Your Low SAT Score

Score 600 on SAT Math

Score 600 on SAT Reading and Writing

Find Your Target ACT Score

Complete Official Free ACT Practice Tests

How to Get a Perfect ACT Score, by a 36 Full Scorer

Get a 36 on ACT English

Get a 36 on ACT Math

Get a 36 on ACT Reading

Get a 36 on ACT Science

How to Improve Your Low ACT Score

Get a 24 on ACT English

Get a 24 on ACT Math

Get a 24 on ACT Reading

Get a 24 on ACT Science

Stay Informed

Get the latest articles and test prep tips!

Follow us on Facebook (icon)

Ellen has extensive education mentorship experience and is deeply committed to helping students succeed in all areas of life. She received a BA from Harvard in Folklore and Mythology and is currently pursuing graduate studies at Columbia University.

Ask a Question Below

Have any questions about this article or other topics? Ask below and we'll reply!

Pardon Our Interruption

As you were browsing something about your browser made us think you were a bot. There are a few reasons this might happen:

  • You've disabled JavaScript in your web browser.
  • You're a power user moving through this website with super-human speed.
  • You've disabled cookies in your web browser.
  • A third-party browser plugin, such as Ghostery or NoScript, is preventing JavaScript from running. Additional information is available in this support article .

To regain access, please make sure that cookies and JavaScript are enabled before reloading the page.

IMAGES

  1. Thesis Statements in APUSH

    how to write a thesis statement for apush dbq

  2. Mr. Walters Writing in APUSH Lesson 1: Making a Thesis

    how to write a thesis statement for apush dbq

  3. APUSH Review: The Introductory Paragraph and Thesis Statement

    how to write a thesis statement for apush dbq

  4. PPT

    how to write a thesis statement for apush dbq

  5. How To Write A Good Dbq Ap World

    how to write a thesis statement for apush dbq

  6. How to Write a Thesis Statement in APUSH

    how to write a thesis statement for apush dbq

VIDEO

  1. Lec 2... How to write Thesis Statement

  2. 7 MINUTE DBQ #4: THE BASIC EVIDENCE POINT #apworld

  3. Thesis Statement| English Essay by Dr Arif Javid

  4. How to write thesis statement in css essay

  5. Emily writes the 2022 APUSH DBQ (National Identity from 1800-1855)

  6. 3 Steps for Teaching THESIS STATEMENTS

COMMENTS

  1. DBQ Thesis Formula (With AP World & APUSH Thesis Examples!)

    If you're not sure how to write a DBQ thesis, check out this post for a failproof DBQ thesis formula and AP World History and APUSH DBQ thesis examples!

  2. AP World History: Sample DBQ Thesis Statements

    Let's take a look at a sample AP World History DBQ question and techniques to construct a solid thesis. Using the following documents, analyze how the Ottoman government viewed ethnic and religious groups within its empire for the period 1876-1908. Identify an additional document and explain how it would help you analyze the views of the ...

  3. The Ultimate APUSH DBQ Guide: Rubric, Examples, and More!

    College Board's APUSH DBQ rubric recommends that you spend 15 minutes reading the documents and 45 minutes writing the essay. The best way to get your time management down is practice. Set timers during your APUSH DBQ practice test so you can get a feel for how much time it takes to put an answer together.

  4. How to Write a DBQ Essay: Key Strategies and Tips

    Start with 20 minutes for your outline and 50 for your essay, (or longer, if you need). Then when you can do it in 20 and 50, move back to 18 minutes and 45 for writing, then to 15 and 40. You absolutely can learn to manage your time effectively so that you can write a great DBQ in the time allotted.

  5. How to WRITE a THESIS for a DBQ & LEQ [AP World, APUSH, AP Euro]

    Resources from Heimler's History: To master all the WRITING SKILLS you need, get my ESSAY CRAM COURSE: +AP Essay CRAM Course (DBQ, LEQ, SAQ Help): https://bi...

  6. How to Ace the AP World History DBQ: Rubric, Examples, and Tips

    How DBQs Work on the AP World History Exam. The DBQ format AP World History uses consists of a single open-ended prompt, and will focus on the time period of 1450-2001. Question Type. # of Questions. % of Total Score. Multiple Choice. 55 questions. 40%. Short Answer.

  7. How to Write the Document Based Question (DBQ)

    Steps to Writing an Effective DBQ. We've summarized how to write an effective DBQ into the following five steps: 1. Read the prompt first. Though you may be tempted to jump into the documents right away, it's very important that you first look at what exactly the prompt is asking for.

  8. PDF 2019 APUSH DBQ Sample Essays by Tom Richey

    All documents are used as evidence to support a claim. The essay makes a complex and nuanced argument supported by strong evidence and analysis that goes beneath the surface. This sample essay was written in order to provide teachers and students with possible approaches to completing the AP US History DBQ.

  9. How to Write a DBQ Essay for APUSH

    As I stated in a previous post on what the APUSH exam is all about, the goal of the exam is to test your historical thinking skills. Historians write arguments based on documents, and for this exam, you will, too. For a DBQ essay, you will receive several documents of varying length. You will be asked to respond to some historical prompt that ...

  10. How to Write a DBQ

    This page details all aspects of writing a DBQ including how to earn the contextualization, thesis, evidence, analysis, and sourcing points, how to write a compare & contrast essay, cause & effect essay, and change & continuity over time (CCOT) essay. It also has a free downloadable worksheet linked to it to help you organize your DBQ.

  11. How to Approach Document-Based AP U.S. History Questions

    During Step 1: Analyze the Prompt. Use the 15-minute reading period to read the documents and organize them into groups for analysis. Feel free to write notes in the test booklet and underline important words in both the source line and the document itself. Nothing in the booklet is read as part of the essay scoring.

  12. APUSH Document Based Questions and Responses: A Study Guide

    Not at all. There could be thousands of things that go under economics, politics, and society - and many things could be considered "unfair" - so I have no idea what the student will be arguing in this DBQ. Thesis Statement Dos and Don'ts. DO: Directly answer the question being asked. Be specific. Write a thesis statement like the ...

  13. PDF THE 2020 APUSH DBQ Template

    THE 2020 APUSH DBQ Template This outline is to help you plan your DBQ on the 2020 AP Exam. You write in the white boxes and your instructions are in the blue boxes. Follow this outline in order and you will be on track to writing a strong essay. ... Shoot for 2-3 sentences PLUS a linkage statement explain how the context relates to the topic ...

  14. PDF Formulating a strong thesis statement for AP History

    he first part of the thesis statement.Basic Complex-split thesis formula:Although (oth. r example) , ultimately (claim responding to prompt specifying 3 examples).Once you get used to writing a complex-split in th. anner, you can tweak the wording - but for now, stick to this formula. Example: Evaluate the main causes of World War 1 Simple ...

  15. How to Earn the Contextualization Point on the APUSH DBQ

    Use the documents and your knowledge of the years 1860-1877 to construct your response. This was the third DBQ we had written, and students were now getting brave enough to move beyond a thesis and document analysis and started attempting to tackle the contextualization point. However, the attempts were all over the map.

  16. The Ultimate Guide to the 2015 AP® US History DBQ

    A good thesis should also respond to all parts of the DBQ prompt. If you write a thesis that succeeds at checking all these boxes, graders will award you your first point. Let us examine what a good thesis statement would look like for the 2015 DBQ and how to avoid writing a bad one. Good:

  17. PDF AP United States History

    Question 1 — Document-Based Question Evaluate the relative importance of different causes for the expanding role of the United States in the world in the period from 1865 to 1910. Maximum Possible Points: 7 . Points Rubric Notes A: Thesis/Claim (0 - 1)

  18. PDF How To Write an AP US History Thesis Statement

    Types of Thesis Statements: 1. Direct: This a straightforward statement that clearly and directly answers the question. To a remarkable degree Jacksonian democrats succeeded in implementing their vision of American society. 2. Compound: Use this approach when trying to prove two main points. Use the word "and.".

  19. United States History Writing Study Skills

    Start with a Clearly Stated Thesis. Some good essay writers begin with a thesis statement, back it up with supporting evidence from documents and outside knowledge, and, if time permits, restate the thesis at the end. Other writers analyze the material and build up logically to their thesis statement. On an AP Exam, you should use whichever ...

  20. Where to Find the Best DBQ Examples

    One of the best ways to prepare for the DBQ (the "document-based question" on the AP European History, AP US History, and AP World History exams) is to look over sample questions and example essays. Doing this will help you to get a sense of what makes a good (and what makes a bad) DBQ response. That said, not all DBQ essay examples are created equal.

  21. PDF AP US History: Writing Introductory Paragraph and Thesis for FRQs OR

    4. Finish paragraph with a clear thesis statement that establishes the purpose of the essay. Example: "Thus, the Civil War did, in fact, represent a political, social and constitutional revolution in America." THESIS TYPES: 1. Simple thesis: "The Civil War was a revolution in American society." (Rating: poor) Thesis is far too simplistic.

  22. PDF AP U.S. History Study Guide: How to Earn a Thesis Point

    o earn this point, yo. r thesis must do three things:1. Respond to the promptYour thesis must be a specific answer to the actual prompt, not a variation of the prompt. or something more generally on the topic of the prompt. This means that you have to read the question multiple. mes and identify everything you must do to address it. It's a.

  23. Prep work for DBQ (pdf)

    World Geography/History Pre-AP Classical Era DBQ Directions: Write a 4 paragraph DBQ Essay using the rubric below as your checklist. Use this packet to breakdown the prompt, analyze the documents, organize your notes, and even draft an introductory paragraph. DBQ Preparation Work 40 pts Completing the DBQ preparation work (pages 2-3 below) will award you 40 points of your major grade.

  24. How To Write a Good Thesis/Opening Statement for DBQ Written ...

    Former teacher/current tutor here! Your thesis should fully reflect the breadth of the documents included in the DBQ. Ideally, three claims for 7 documents; each claim is a subargument that frames each paragraph. You need to be fully aware of what kind of thinking skill you're being asked to use, and ensure that your thesis fully answers the ...